PNCBExam3PNP

¡Supera tus tareas y exámenes ahora con Quizwiz!

A toddler who attends daycare has diarrhea that began with three days of low-grade fever and foul smelling watery stools. One month later, loose stooling continues and there is now abdominal distention. The MOST probable causative organism is: A. Giardia duodenalis B. norovirus C. rotavirus D. Shigella toxicum

The correct answer is A - Giardia duodenalis Giardia duodenalis (formerly Giardia lamblia and Giardia intestinalis) is a flagellate protozoan found in contaminated water sources and food. It is the most common parasitic infection in the United States and is commonly transmitted in daycare centers. Symptoms typically range from mild to moderate in children and can include abdominal cramps, flatulence, bloating, and watery, greasy foul-smelling stools. The incubation period is 1-3 weeks, and the duration of illness is typically prolonged. Some infections are self limited while others may require antimicrobial treatment. Rotavirus presents with acute fever and vomiting. Watery diarrhea occurs 2-4 days later in children less than 5 years of age. The duration of illness is 3-8 days with a short incubation period of 48 hours. It is a self-limiting virus that occurs more commonly in cooler months. Treatment is supportive with replacement of fluids and electrolytes. The advent of the rotavirus vaccine in 2006 has decreased the burden of the disease dramatically. Infection with Shigella begins with fever, abdominal cramping, and diarrhea. It is most common in children 6 months to 3 years of age, with an incubation period of 24-48 hours and a duration of illness of 4-7 days. The majority of cases are self-limiting and do not require antimicrobial therapy with the mainstay of treatment being supportive correction of fluid and electrolyte imbalances. Abrupt onset of watery diarrhea, nausea, vomiting, and abdominal cramping are the symptoms of Norovirus; the incubation period is 12-48 hours, and the duration can be 24-60 hours. Transmission of this virus is most common in the cooler months. Treatment is also supportive to maintain fluid and electrolyte balance.

For which of the following foreign bodies is endoscopic removal MOST emergent? A. a single button battery in the esophagus B. a single cylindrical battery in the esophagus C. two packets of undetermined drugs in the stomach D. two magnets, one in the stomach and one in the esophagus

The correct answer is A - a single button battery in the esophagus Foreign body ingestions occur most commonly in children between the ages of 6 months to 3 years as well as in older children with developmental delays. Intentional ingestions are more frequent in adolescents, adults, and in those with underlying behavioral or mental health diagnoses. Interventions for ingestions are based on the object or objects ingested, time since ingestion, anatomic location of the object, and presenting symptoms. Any foreign body lodged in the esophagus is considered an emergency due to the risk of perforation and sepsis. Button battery ingestions have the greatest potential for serious injury. Lodged in the esophagus, a button battery can result in burns and liquefaction necrosis to tissues within 2 to 3 hours of ingestion. An ingested button battery lodged in the esophagus is therefore the most emergent indication for endoscopic removal. A single ingested button battery which reaches the stomach carries a lower risk, but time since ingestion is an extremely important factor as there may have been adequate exposure within the esophagus to cause severe tissue injury. An algorithm for management of button battery ingestions has been published and adopted by the National Capital Poison Center. It can be accessed on their website at www.poison.org. Cylindrical batteries have been associated with fewer and less serious complications than button batteries, if ingested. There is limited literature regarding ingestion of cylindrical batteries and no algorithm for management of this ingestion. Removal of single cylindrical batteries from the esophagus is typically done within 24 hours. Once these batteries reach the stomach, observation for passage in the stool is appropriate. If multiple cylindrical batteries are ingested, endoscopic removal is typically recommended. Injury from ingestion of high-powered magnets has been the topic of increased literature within the past decade; including management guidelines primarily based on expert opinion in 2012 provided by the North American Society for Pediatric Gastroenterology, Hepatology, and Nutrition (NASPGHAN) Endoscopy Committee. 1 Ingestion of multiple high-powered magnets, or magnets and metallic objects simultaneously, can result in pressure necrosis of the bowel wall and ischemia or fistula if the magnets or magnet and metallic objects attract across tissues. For these reasons, multiple magnet ingestion should be closely monitored, usually inpatient, with x-rays. Consult with a pediatric gastroenterologist and pediatric surgeon is warranted. Ingestion of a single magnet is usually benign after it arrives in the stomach, with the majority passing without incident if no further batteries or metallic objects are ingested or worn on the body or clothes. A single magnet lodged in the esophagus is treated as any blunt object and removed endoscopically within 24 hours if it does not pass into the stomach. Ingesting packets containing drugs is primarily done for purposes of transporting illegal substances without detection. Endoscopic removal of these packets is not indicated and is avoided due to risk of rupturing the package and causing exposure to potentially fatal amounts of drug. If bowel obstruction or perforation is suspected, immediate surgical intervention is indicated. In those who are asymptomatic, use of laxatives and bowel irrigation has been described. Toxidrome symptoms, if present, are treated until the packet is passed or surgically removed.

An otherwise healthy 11 month old is diagnosed with bronchiolitis on the 3rd day of illness. The MOST important caregiver education includes instructions to A. call the office if the infant is having less than 6 wet diapers in 24 hours. B. take the infant to the emergency department if fever continues for more than 48 hours. C. take the infant to the emergency department if wheezing continues for more than 5 days. D. call the office if the infant's cough continues for more than 1 week.

The correct answer is A - call the office if the infant is having less than 6 wet diapers in 24 hours. Respiratory syncytial virus (RSV) is the most common etiology of bronchiolitis and pneumonia in children under 1 year of age. For children under the age of 6 months, 1-2% will require hospitalization. 1 In developed countries, the fatality rate is <1%, with deaths attributable to respiratory arrest/failure or severe dehydration and electrolyte disturbances. 2 Higher mortality rates are seen in children with comorbidities such as congenital heart disease and immunodeficiency, as well as in those with complex medical needs. 2 In infants younger than six months, RSV infection may result in symptoms of irritability, poor feeding, lethargy, and/or apnea with or without fever. In older infants and young children, rhinorrhea and decreased appetite may appear one to three days before cough, often followed by sneezing, fever, and sometimes wheezing. 1 Paroxysms of coughing may trigger vomiting, that along with poor oral intake and tachypnea, increase the risk of dehydration. Thus, the hydration status of an infant with bronchiolitis needs to be carefully monitored. A typically well-hydrated infant will have at least 6 wet diapers per 24 hours. The first 48 to 72 hours after the onset of cough are the most critical. The child can be ill-appearing but gradually improves over a period of several days. Wheezing is expected for more than 5 days and a fever of up to 102 F (38.9 C) is common in the first 3-7 days of illness, which further contributes to the risk of dehydration. Coughing commonly continues for 2-3 weeks and may cause the infant to vomit on occasion, also affecting the infant's hydration

Parents are concerned that their 4 month old is not sleeping through the night and is waking after 5 hours of sleep. Which of the following is the appropriate anticipatory guidance? A. explain that this is typical sleep behavior for an infant this age B. offer reassurance that infants start to sleep through the night by 5 months of age C. suggest feeding the infant a serving of baby cereal before bedtime to offset hunger during the night D. advise that infants with irregular sleep patterns at this age may have disrupted sleep until 1 year old

The correct answer is A - explain that this is typical sleep behavior for an infant this age Infants who are 4 - 5 months of age typically sleep about 14 hours a day, which includes an average of about 4.5 hours of daytime napping taken over 2 - 3 naps. There is variability in sleep patterns and sleep regulation among infants, although by 3 months of age sleep consolidation increases with longer sleep times during the night. Transitioning to sleep is aided by the infant's ability to self-soothe that typically develops in the first 3 months. The variability in sleep patterns among infants was reviewed in a study of 388 infants at 6 months old and 369 infants at a year old. At 6 months of age, 38% of typically developing infants were still not sleeping 6 consecutive hours at night; however, by 12 months of age that number had decreased to 28%. Study authors referred to the idea of infants sleeping through the night at 6 to 12 months of age as a "gold standard" pervasive in Western nations, and further suggested that more education about normal development and variability in sleep-wake cycles would benefit parents. The American Academy of Pediatrics healthchildren.org website addresses infant sleeping and advises parents that, "...a good sleeper is a child who wakes up frequently but can get himself back to sleep. It is not a child who sleeps without waking for 10 hours at night. Frequent waking is developmentally appropriate and allows the baby to wake up when he is in a situation in which he is not getting enough oxygen or is having problems breathing. Sleeping undisturbed for prolonged periods at this age is not healthy." While it is recommended that infants 4 - 12 months of age should sleep 12-16 hours a day to include naps, that sleep is not contiguous. Four-month-old infants must wake through the night and feed to ensure they receive adequate calories. At 4 months of age, breastmilk or formula remains the best food for infants. The AAP recommends delaying the introduction of solid foods until 6 months old. At this age, parents can influence sleep patterns by maintaining somewhat consistent daily routines for feeding, napping, and bedtime practices. The healthcare provider has an important role in offering caregiver guidance to promote realistic expectations regarding infants' sleep-wake cycles. It is also helpful to discuss ways to respond to infants during night awakenings that encourage an infant's return to sleep.

An adolescent presents with a one-week history of fever, exudative pharyngitis, cervical lymphadenopathy, and is noted on exam to have hepatosplenomegaly. Which of the following laboratory studies is MOST appropriate to order? A. antistreptolysin O titer B. complete blood count C. cytomegalovirus titers D. heterophile antibody test

The correct answer is D - heterophile antibody test This adolescent's symptoms raise suspicion for mononucleosis. The most appropriate laboratory test to order would be a heterophile antibody test (monospot). This is a nonspecific test and is positive in 85% of adolescents with mononucleosis within the first two weeks of the illness but is often negative in children under the age of 4 years. A complete blood count does not provide any specific diagnostic information. The antistreptolysin O titer tests for recent strep infection, but these symptoms are not consistent with strep. Cytomegalovirus (CMV) is a good differential diagnosis as it can cause fatigue and adenopathy, but the presence of hepatosplenomegaly and pharyngitis is more specific for mono. The most common cause of mono is Epstein-Barr virus (EBV). Epstein-Barr titers can be used for preliminary mono testing if heterophile antibody testing is negative and there is still strong suspicion for mononucleosis, or in young children in which heterophile antibody tests are unreliable.

A toddler presents with a one-week history of fever, diarrhea, and malaise. The caregiver notes that several other children in daycare have had similar symptoms. Physical examination reveals icteric sclera and mild hepatomegaly. The PRIORITY diagnostic test is: A. anti-HAV IgM B. complete blood count C. hemoglobinopathy study D. hepatic ultrasound

The correct answer is A - anti-HAV IgM Hepatitis A is a primary infection in the liver. It is a highly contagious virus spread by person-to-person contact and fecal-oral contamination of food and water. Symptoms of hepatitis A infection begin with acute febrile illness, often associated with malaise and gastrointestinal complaints. Jaundice develops within one week and may initially be noticed in the sclera or it may only be found in the sclera. Hepatitis antibodies are present at the onset of symptoms, making serum anti-HAV IgM a valuable diagnostic tool. Hemoglobinopathies can cause jaundice but are not associated with febrile illness. Hepatic ultrasound and complete blood count may be indicated but would not be the priority diagnostic tests given the clinical presentation.

Caregiver education for an infant with croup includes that urgent medical re-evaluation is needed if the infant has: A. audible stridor when calm B. barky cough in the morning C. fever of 102 F (38.9 C) or higher D. retractions between ribs with crying

The correct answer is A - audible stridor when calm Croup is a common illness in children younger than 6 years of age with a predilection for infants and toddlers 6 to 36 months of age. Croup can be caused by viruses and allergens. Presentation includes fever, rhinorrhea, hoarse "barking" cough, upper airway stridor, retractions that range from mild to severe, and an increased respiratory rate. Symptoms usually worsen at night including the cough and stridor, which can be especially frightening to parents and caregivers. Hypoxia occurs as a very late sign. A fever of up to 102 F (38.9 C) may be part of croup or an indication of a more severe illness but, in isolation, does not warrant urgent re-evaluation. Severity progresses from symptoms that are present when crying, active, or upset, to symptoms present while at rest. A barky cough in the morning is a common feature of croup.

INITIAL treatment for a child with amblyopia and a droopy left eyelid includes: A. correction of the ptosis B. lubrication of the eye at night C. atropine solution in the left eye D. apraclonidine solution in the right eye

The correct answer is A - correction of the ptosis Amblyopia is a problem that can have many causes and leads to loss of vision. Types of amblyopia are: deprivation amblyopia: results from occlusions of the visual axis, refractive amblyopia: occurs when a child's refractive errors are not similar and vary by a wide margin, strabismic amblyopia: results from misalignment of the visual axes. Amblyopia can be corrected when identified early, ideally before 5 - 7 years of age, and if the inciting factor is corrected. Amblyopia is suspected when a preverbal child resists having the eye with good vision covered or restricted. Additionally, when the pupil is consistently covered due to conditions such as ptosis, persistent stye, or a hemangioma in the lid area, amblyopia should be considered as a consequence. Treatments for amblyopia include patching and pharmacologic blurring of the unaffected eye using atropine ophthalmic drops. However, if there is occlusion of an eye, as in the case of ptosis, this must be corrected before any other treatment will be beneficial. Apraclonidine (Iopidine) ophthalmic solution is used to confirm the lack of sympathetic input in Horner syndrome. Lubrication of an affected eye at night is used to protect the eye from injury when there is incomplete closure of the eyelids.

In adolescents with ADHD, providing for adequate control of symptoms is MOST important to address when they will be A. driving independently. B. attending social events. C. participating in competitive sports. D. taking advanced placement courses.

The correct answer is A - driving independently. The Clinical Practice Guideline for the Diagnosis, Evaluation and Treatment of ADHD in children and adolescents was updated in 2019. Little has changed in the guidelines from the 2011 publication. The 2019 guideline includes the addition of a key action statement to evaluate for co-morbid emotional, behavioral, and developmental conditions. When young children are diagnosed with attention deficit / hyperactivity disorder (ADHD), the primary concern is often academic success. However, those concerns are compounded with safety risks as the adolescent with ADHD begins to become more independent. In addition to the inherent risks of driving, there is also an increased risk of distractibility in an adolescent with untreated ADHD, which could lead to motor vehicle collisions. Special concern should be taken to learn about an adolescent's activities and provide medication coverage for a full day, especially if part of the afternoon or weekend activities includes driving a motor vehicle. Longer acting medications or short acting medications taken in the late afternoon might be helpful. Impairment from ADHD can continue into adulthood. Longitudinal studies show that individuals with ADHD have an increased risk of early death, suicide, associated substance use disorders, and increased rates of criminality.

According to the American Academy of Pediatrics, non-medical exemptions for vaccination should be A. eliminated completely. B. allowed for both religious and personal beliefs. C. eliminated for any reason other than religious beliefs. D. allowed only as a delayed or alternative schedule in areas with immunization rates greater than 80%

The correct answer is A - eliminated completely. Vaccinations have significantly reduced the incidence, prevalence, morbidity and mortality of a number of communicable diseases in the Western world. It is estimated that administration of the recommended childhood immunizations for an annual U.S. birth cohort prevents approximately 20 million illnesses and over 40,000 deaths and saves nearly $70 billion in costs. Proof of immunization is required by law for child care and school attendance in all U.S. states and the District of Columbia. All states and the District of Columbia provide for exemption from required immunization for medical reasons, such as allergy to or a history of a significant reaction to a vaccine or its components, history of a significant reaction to a vaccine, and underlying health conditions such as immunosuppression. Additionally, most states allow exemptions from vaccinations based on religious beliefs, and almost half allow exemptions based on philosophical or personal beliefs. Evidence supports that higher rates of vaccine exemptions correlate with higher rates of vaccine-preventable illnesses and disease outbreaks. Herd immunity, also known as community or population immunity, provides indirect protection for unimmunized individuals by decreasing the incidence of vaccine-preventable illnesses and their subsequent spread in communities with sufficient numbers of immunized individuals. Unvaccinated or incompletely vaccinated children are at greater risk for contracting vaccine-preventable diseases. Children who are exempted from vaccination for medical reasons increase the number of susceptible individuals in a population, thus contributing to decreased herd immunity. The American Academy of Pediatrics (AAP) supports legal requirements for immunization for childcare and school attendance, and medically indicated exemptions for specific vaccines for individual children. The referenced clinical report states that the AAP, along with the American Medical Society and the Infectious Diseases Society of America, supports elimination of all nonmedical exemptions for childhood vaccines. As stated in its policy statement, the AAP regards nonmedical vaccine exemptions as inappropriate for individual, public health, and ethical reasons.

An adolescent is about to start driving to school with friends. Which of the following interventions is the MOST important to promote the adolescent's safety? A. encourage parents and adolescent to develop a contract addressing safe ride situations B. ask the adolescent to demonstrate knowledge of safe driving laws to promote parental confidence C. encourage adolescent to provide parents with friends' phone numbers to foster trust D. provide pamphlet highlighting interviews with teens about how unsafe driving affected their lives

The correct answer is A - encourage parents and adolescent to develop a contract to address safe ride situations Driving represents a rite of passage for adolescents and a level of independence. However, risk factors such as inexperience, immaturity, vulnerability to distraction, and a developmental propensity for underestimating risk contribute to drivers between the age of 15-19 years having the highest rate of motor vehicle collisions. The AAP recommends parents make a plan with their adolescent for situations when he/she feels unsafe driving or riding with someone. Formalized agreements, such as the Contract for Life created by the Students Against Destructive Decisions (SADD) can be included in the plan. Previous research by Dr. John Knight and Dr. Sion Kim Harris revealed that teens who received a Contract for Life at their pediatrician's office were at a significantly lower risk for riding in a car with a driver who had been using alcohol or drugs compared to patients who did not receive a contract. Guidance to caregivers should include the impact of their own role modeling on the driving practices of their adolescent drivers. Research supports that teens who observed a parent driving after drinking were three times more likely to drive after drinking that those whose parents modeled safe behaviors.

An adolescent male with history of unprotected heterosexual intercourse presents with a red, swollen, and painful scrotum. His pain is reduced by elevation of the scrotum. What is the MOST likely diagnosis? A. epididymitis B. inguinal hernia C. torsion of teste D. varicocele

The correct answer is A - epididymitis Epididymitis, or inflammation of the epididymis, occurs most frequently in late adolescence and early adulthood, usually due to bacterial infection. In heterosexual men, the most common etiologies are gonorrhea and chlamydia; in males who practice insertive anal sex, it may additionally be caused by enteric organisms such as E. coli. Epididymitis presents typically with a gradual onset of redness, swelling of the scrotum, and a tender, indurated, and hard epididymis. Pain from epididymitis may be relieved with elevation of the scrotum, while in testicular torsion the symptoms present more acutely, and elevation of the scrotum increases the pain. Varicoceles are enlarged vessels causing a painless mass in the scrotum that often is described by the patient as feeling like "a bag of worms;" with prolonged standing activity, there can be increased dull aching pain which is relieved when the individual reclines. An inguinal hernia presents as a swelling in the inguinal area or scrotum containing abdominal contents passing through a weakness in the abdominal wall. A painful hernia is typically incarcerated, while a non-incarcerated hernia can be reduced.

For an adolescent with special needs or chronic illness the MOST common barrier related to transitioning health care is A. finding an adult healthcare provider for transition. B. lack of healthcare provider time to plan for transition. C. difficulty in talking with patients about transitioning care. D. resistance of the family and adolescent to transition of care.

The correct answer is A - finding an adult healthcare provider for transition. Finding an adult healthcare provider who is qualified to care for young adults with special health care needs is the most commonly perceived barrier to the successful transition of health care as identified by family and young adults, pediatric healthcare providers, and adult internists. Transitioning of care requires time and communication with the parents and adolescent involved. Many families may be hesitant to leave the nurturing environment of pediatric care and may perceive that adjusting to an adult practice as difficult. Internists may lack the training and qualifications to address many of the complicated health care needs of adolescents with chronic illnesses. Because of the delicate nature of such conversations, some pediatric providers may not be comfortable in dealing with the complexities of transitioning care.

A preschooler who was adopted internationally has a history of a positive purified protein derivative (PPD) skin test. Which of the following is MOST important to ascertain FIRST? A. history of Bacille Calmette-Guérin (BCG) vaccination B. chest radiograph result C. age at positive PPD D. size of reaction

The correct answer is A - history of Bacille Calmette-Guérin (BCG) vaccination Almost 80% of all childhood tuberculosis (TB) is due to contact of the child or a household member with an individual from a foreign country who has an active infection. Bacille Calmette-Guérin (BCG) immunization can produce a positive tuberculin skin test result. BCG is a live attenuated vaccine that, while rarely used in the United States, is one of the most widely used vaccines in the world. Determining if a positive tuberculin skin test (TST) is due to a prior BCG vaccination or due to true TB disease requires the use of interferon gamma release assay (IGRA) testing such as QuantiFERON-TB Gold In-Tube (QFT) and T-SPOT.TB. Recommendations for specific methods for testing have subtle variances that include indications for use based on age. The "Guidance on Strategy for Use of TST and IGRA by age and BCG-Immunization Status" provided in the AAP Red Book 31st ed. (page 836, FIG 3.11) addresses indications for the use of interferon gamma release assay (IGRA) for children who are less than or 2 years of age who have had previous vaccination with BCG.

Which of the following signs or symptoms is MOST often associated with anorexia nervosa? A. low body temperature B. salivary gland enlargement C. elevated erythrocyte sedimentation rate D. low thyroid stimulating hormone (TSH) level

The correct answer is A - low body temperature Disordered or dysfunctional eating occurs along a spectrum. Both anorexia nervosa and bulimia nervosa are characterized by a distortion of body perception, often accompanied by depression. Anorexia nervosa's physical signs and symptoms are related to chronic malnutrition; hence amenorrhea, hypothermia, hypotension, and bradycardia are often seen. Bulimia nervosa involves episodes of binge eating and compensatory behavior to prevent weight gain, often purging. If the purging behavior involves vomiting, salivary gland enlargement can be present.

A preschooler is evaluated for a history of fever and intermittent bone pain and is noted to have an abdominal mass. Which is the MOST likely diagnosis? A. neuroblastoma B. non-Hodgkin's lymphoma C. osteogenic sarcoma D. Wilms tumor

The correct answer is A - neuroblastoma Neuroblastoma is the most common extracranial solid tumor in children and the most commonly diagnosed malignancy in early childhood. The median age at diagnosis is 22 months, and approximately 90% of cases have been diagnosed by 5 years of age. Neuroblastoma may develop at any site of sympathetic nervous system tissue with over half developing in the adrenal glands. Symptoms may mimic many other disorders and can be hard to diagnose. Metastasis of neuroblastoma typically occurs in children over 1 year of age and most commonly includes the lymph nodes, long bones and skull, bone marrow, liver, and skin. Signs and symptoms reflect the tumor site and extent of disease, but may include fever, failure to thrive, bone pain, cytopenia, orbital proptosis, masses, bowel obstruction, or spinal cord compression. Wilms tumor is an embryonal malignancy of the kidney that can also present with an abdominal mass. Sarcomas of the extremities are more likely to occur in older children.

When used in treatment of acne vulgaris, topical retinoids act to A. normalize desquamation of the follicular epithelium. B. minimize development of antibiotic-resistant Cutibacterium acnes (formerly Propionibacterium acnes). C. generate free radicals to oxidize proteins in the cell wall of Cutibacterium acnes. D. work within the cell ribosomes to alter upstream signals affecting inflammatory pathways.

The correct answer is A - normalize desquamation of the follicular epithelium. The therapeutic objectives in the management of acne vulgaris are to reduce sebum production, prevent formation of microcomedones, suppress Cutibacterium acnes (formerly Propionibacterium acnes), and reduce inflammation to prevent scarring. Topical retinoids, vitamin A derivatives that bind to retinoid receptors in the skin, are a key component of primary acne management. Topical retinoids normalize desquamation of the follicular epithelium to prevent new formation of microcomedones, the precursors of both comedonal and inflammatory lesions. Additionally, retinoids promote clearing of existing microcomedones and work within the nucleus to alter downstream signals that affect inflammatory pathways. The evidence-based recommendations for the diagnosis and treatment of pediatric acne (last published in 2013) support the use of topical retinoids as monotherapy or in combination with other agents for all types and severities of acne in children and adolescents.

For a 2 day old who has spitting and choking with feedings, which procedure is anticipated as part of an evaluation before hospital discharge? A. placement of an orogastric feeding tube with chest/abdominal radiograph B. placement of a nasogastric feeding tube and fluoroscopic evaluation C. CT of the esophagus and stomach with oral contrast D. barium swallow with small bowel follow through

The correct answer is A - placement of an orogastric feeding tube with chest/abdominal radiograph Esophageal atresia (EA) with or without tracheoesophageal fistula (TEF) is commonly diagnosed in the newborn period when an infant has recurrent spitting and choking with feeds. The diagnostic test for EA is the passage of an orogastric tube followed by radiologic imaging to confirm coiling of the tube in the thoracic region. Upper GI studies are often used to diagnose obstruction, intestinal malrotation, and intestinal duplication. CT scan would deliver potentially unnecessary radiation and is not needed for diagnosis, nor is a nasogastric tube with fluoroscopy. Newborns are obligatory nose breathers, so it is best not to put the feeding tube in the nose.

A mother is concerned about a lump identified under the nipple of her 10-year-old daughter's breast. The child recently completed treatment for leukemia. On examination the right nipple is elevated with soft tissue that is non-tender to palpation. The MOST appropriate next step is to A. reassure that this is normal breast development for the child's age. B. obtain bloodwork to assess hormone levels and blood counts. C. arrange an ultrasound to assess the swelling. D. consult with the child's oncologist.

The correct answer is A - reassure that this is normal breast development for the child's age. Initial breast development usually begins with unilateral disc-like sub-areolar swelling. Many adolescents and their parents present with concerns of breast tumors at the time of initial development. Breast budding is expected between 9-10 years of age and occurs in 97% of females by age 12, which supports this as a normal finding. Parents of childhood cancer survivors are concerned about relapse and second malignancies. Reassurances about normal growth and development are often needed to minimize anxiety. Ultrasound and blood work are unnecessary, as is consulting oncology.

When completing a physical assessment of a 12 month old male's genitalia, which finding warrants further evaluation? A. reducible hydrocele B. inability to retract foreskin of penis C. testis needing to be milked into the scrotum D. presence of smegma under foreskin of penis

The correct answer is A - reducible hydrocele Physical assessment findings of the male genitalia that may be normal at one age may be abnormal at another age. The inability to retract the foreskin of the penis is normal until about 6 years of age. It is common for the testis to retract during an examination or to cold or touch in infancy and early childhood. In this age group placing a finger above the inguinal canal and "milking down" the testis into the scrotum prevents this retraction. Medical terminology for the whitish matter under the foreskin of the penis is smegma, and it is a normal body substance. A hydrocele is an enlargement of the scrotal sac containing serous fluid. A hydrocele can further be classified as non-communicating or communicating. In a noncommunicating hydrocele, the scrotum is tense and non-reducible, the fluid does not fluctuate, and the hydrocele will resolve without intervention. A communicating hydrocele is reducible, has a fluctuating fluid volume, and is more often associated with hernias. Many communicating hydroceles that occur in infancy will resolve without surgery; however, if they persist at 1 year, referral for surgery is warranted.

A caregiver who is primarily Spanish-speaking reports during a visit that her 24 month old is bilingual and speaks four words. Which of the following is the BEST initial management? A. refer to a pediatric speech pathologist with access to translators B. encourage caregiver to choose one language for communication C. advise the caregiver to read to the child daily D. refer to audiology for a hearing evaluation

The correct answer is A- refer to a pediatric speech pathologist with access to translators Language and speech development are critical areas to address at preventative exams. At two years of age a child should be speaking short phrases of two to three words with a vocabulary of about 50 words, of which 25% are intelligible to strangers. Children from bilingual homes or who are bilingual tend to develop language more slowly, mixing words and phrases from the two languages; however, milestones should continue to fall within expected ranges. Bilingual children with significant delays in vocabulary require the same evaluation as monolingual children with language delays. The first step in management for children with delayed vocabulary is referral to a pediatric speech pathologist with access to translators. Having older siblings monitor speech at home is not culturally appropriate and does not address the needs of the child. Daily reading does help with language development and should be a part of the anticipatory guidance at this age; however, this child is showing signs of language delay. A hearing evaluation is an appropriate element of assessing speech and language delays, but the initial step is referral to a pediatric speech pathologist.

Which of the following is the BEST INITIAL counseling for a family of a 5 year old with primary nocturnal enuresis? A. symptoms are often a developmental lag that will be outgrown B. wake the child frequently during the night so urination is effective C. alarm systems are easy to use but less effective than medication therapy D. treatment with desmopressin has been effective with low rates of relapse

The correct answer is A- symptoms are often a developmental lag that will be outgrown Primary nocturnal enuresis is a common problem in children between the ages of 5 and 8 years. In most cases, the symptoms will be outgrown without treatment. Once any medical or physical problem is ruled out, common sense approaches include limiting fluids after dinner, encouraging urination right before sleep, and holding the urine for longer periods during the day. These procedures do help in many cases and should be the first counseling information given to families. Active treatment should not occur before 6 years of age, and punitive consequences are avoided to prevent harm to the child's self-esteem. Alarm systems work well in waking the child when the first few drops of urine are sensed and are considered first line treatment. Alarm systems have lower relapse rates than medications. Medications like DDAVP and imipramine are also effective as long as they are used regularly, but relapses can occur when they are stopped.

The developmental assessment for which of the following children is MOST concerning for cerebral palsy? A. 3 month old who has a mild head lag when pulled to a sitting position B. 5 month old who pulls to standing from supine position C. 7 month old who briefly maintains sitting position D. 8 month old who jumps up and down when held in a standing position

The correct answer is B - 5 month old who pulls to standing from supine position Developmental surveillance occurs at every visit and includes listening to the concerns of the caregiver and the provider's observations of the child's development. Standardized development tools such as the Ages and Stages Questionnaire (ASQ) are formal, validated screening tools that score the child's abilities and are used by the healthcare provider to identify developmental delays. The AAP recommends screening with a developmental tool at 9, 18, and 30 months or when surveillance reveals a concern that warrants more information. Abnormalities in development may fall into categories such as milestones achieved too early, achieved later than expected, persistence of milestones, or incorrect completion of the milestone. A 5 month old who pulls to a standing (rather than sitting) position from supine is demonstrating muscle rigidity which is a concern for cerebral palsy. The other gross motor findings are within a normal expected timeframe for performance.

An exclusively breastfed newborn whose birth weight was 7 lbs (3.175 kg) had a discharge weight at 2 days of age of 6 lbs 11 oz (3.033 kg). Using an expected weight gain per day, the minimum expected weight at one week of age is: A. 6 lbs 12 oz (3.062 kg) B. 6 lbs 13.5 oz (3.104 kg) C. 6 lbs 14.5 oz (3.133 kg) D. 7 lbs (3.175 kg)

The correct answer is B - 6 lbs 13.5 oz (3.104 kg) Normal newborn infants lose 5% to 10% of their birth weight in the first few days of life. It is helpful for parents to be aware of both the birth and discharge weights. The minimum expected weight gain is 0.5 oz/day (14.2 g/day), and thus for 5 days a 2.5 oz (921.4 g/day) gain is minimally expected. For breastfed infants, once the maternal milk volume increases, the infant begins to gain weight in the range of 0.5 to 1 oz/day (14.2 to 28.4 g/day) or 4 to 7 oz/week (113.4 to 198.4 g/week). By 2 weeks of age, many breastfed infants have regained their birth weight, although others may take up to 3 to 4 weeks. Any gain less than 0.5 oz/day (14.2 g/day) would signal a concern for poor breastfeeding or the potential for a serious disease process.

The MOST likely diagnosis for a child who is being evaluated for a recent onset of fever, weight loss, poor appetite, abdominal pain, diarrhea, and recurrent aphthous ulcers is: A. acute gastroenteritis B. Crohn disease C. enterocolitis D. pseudomembranous colitis

The correct answer is B - Crohn disease Fever, diarrhea, and abdominal pain in children can range from a short-lived viral illness to more urgent surgical conditions. The initial presentation of a chronic illness should also be considered. Most children with Crohn disease present with the classic symptoms of diarrhea, abdominal pain, and weight loss. Chronic perianal disease including skin tags, fissures, fistulae, and abscesses may be present. Recurrent aphthous stomatitis is also suggestive of Crohn disease. Poor appetite, fever, and iron deficiency anemia are also commonly noted at initial presentation. A child with acute gastroenteritis often presents with vomiting, diarrhea, decreased appetite, and occasionally low-grade fever. Enterocolitis is inflammation of the small and large intestines typically presenting with fever, abdominal distention, and bloody stools. Symptoms of pseudomembranous colitis can start after a couple of days of taking an antibiotic and include blood or pus in diarrheal stools, along with abdominal pain and fever.

A caregiver states she has been giving her 18 month old's hand a slap when the toddler is rough with the family dog, but that it does not seem to be correcting the behavior. Which of the following is the BEST response? A. Suggest explaining to the toddler that the actions are hurting the dog. B. Discuss toddler's developmental abilities and advise saying "no" while removing child from environment. C. Explain that because toddlers lack a sense of morality, they are unable to understand why behavior is wrong. D. Inform the caregiver that while physical punishment can be effective there is a risk of psychological harm.

The correct answer is B - Discuss toddler's developmental abilities and advise saying "no" while removing child from environment. Discipline is education. Its purpose is to provide expectations to the child for appropriate behavioras opposed to punishment, which is administered in response to behavior and typically involves pain or suffering. In this scenario, the best option is to control the child's environment by removing the toddler from the situation and giving a verbal "no." It is important to help the caregiver understand that consistency will be key and that developmentally the toddler is unable to understand that the behavior is wrong. A toddler's sense of morality is primarily associated with pleasing the caregiver rather than through their own internal controls, which do not develop until the preschool years. Slapping is a form of punishment that only serves to retaliate against the undesired behavior rather than teaching to correct the behavior. There is evidence that corporal punishment can lead to an increased risk of negative behavioral, cognitive, psychosocial, and emotional outcomes for children and, therefore, it's use is not recommended by the AAP. As stated in the AAP's 2018 policy statement about effective discipline, "...although many children who were spanked become happy, healthy adults, current evidence suggests that spanking is not necessary and may result in long-term harm".1 When explaining this to adults who care for children, it is essential to also include alternative forms of discipline. Recommendations regarding healthy forms of discipline include using positive reinforcement of appropriate behaviors, limit setting, redirecting, and providing clear behavioral expectations.

While some characteristics of an Individualized Education Plan (IEP) and 504 Plan are similar, the IEP differs in that it uniquely requires A. a specific medical diagnosis for eligibility. B. a modification in curriculum to meet the student's needs. C. an evaluation typically completed through school system resources. D. a written plan be developed that specifies individualized accommodations.

The correct answer is B - a modification in curriculum to meet the student's needs. For children with special healthcare needs, much of their day-to-day care occurs outside of the medical setting. It is important that providers are familiar with the nonmedical and community resources that are available to these children and their families. The school setting provides many educational, developmental, and psychosocial support services, and providers need to be prepared to help parents access and utilize these resources. For school-age children, the Individualized Education Plan (IEP) and 504 Plan can both help ensure they receive services needed to optimize their educational experience. The overlap between these two plans can make it difficult to determine which is most appropriate for a child's specific needs. Eligibility for both plans requires a medical diagnosis. For a 504 Plan, this can be any psychologic or physical disorder which "substantially limits" the individual's learning. These children can follow the curriculum without modification but may require assistance in the school setting. This may include conditions such as ADHD, diabetes, or migraines. In order to be eligible for an individualized education plan (IEP), a child has to also have a medical diagnosis, but that diagnosis must specifically meet criteria as a qualifying disability that seriously affects learning or behavior and, therefore, requires modification of the school curriculum. Both a 504 and an IEP require evaluation, although the evaluation for an IEP is more formalized and is typically completed by a team of professionals. Both an IEP and a 504 plan will include a written plan. There are no legal requirements for what is included in a 504 plan and there is no mandated re-evaluation. An IEP has very specific criteria and must describe the specific learning problems, detail what services will be provided, set annual goals and define how progress will be measured. Re-evaluation is mandated every 3 years.

A school-age child who has had a few prior episodes of wheezing, presents with 3-days of URI symptoms to include coughing at night, rapid breathing, and loud wheezing evident on both inspiration and expiration. In the office, a short-acting B2-agonist is administered and there is some improvement; however, mild expiratory wheezing persists. Which action is the BEST NEXT step? A. refer to emergency department B. add oral systemic corticosteroid C. continue inhaled albuterol every 2 hours D. schedule follow-up appointment next week

The correct answer is B - add oral systemic corticosteroid While goals for asthma management include preventing exacerbations, flare-ups may occur. Treatment of flare-ups focus on both immediate management and the overall evaluation of the preventative maintenance strategies. Initial management of an asthma flare is the reversal of the bronchoconstriction with the use of inhaled albuterol (short-acting B2-agonist (SABA)) via metered dose inhaler or nebulizer treatment, with up to two treatments given every 20 minutes. The response to the SABA medication is then determined to be good, incomplete, or poor. Follow-up to a good response involves continued use of SABA every 3-4 hours for 24-48 hours and a consideration of a short course of oral systemic corticosteroids. For the those with an incomplete response, SABA is continued every 3-4 hours and oral corticosteroids are added with follow-up contact with the healthcare provider. For those with a poor response, oral corticosteroids are added, SABA is repeated, and a same day appointment is recommended if the triage and advice has been given by phone. For those with severe distress or nonresponse, an evaluation at the emergency department is needed with SABA being administered en route.

When discussing confidentiality with an adolescent prior to obtaining a developmental screening and assessment, which of the following information is appropriately included? A. provider's obligation to keep any information that is shared confidential B. adolescent's responsibility for deciding what information can be shared with others C. sensitive health issues need to be shared with parents/caregivers or another trusted adult D. importance of parents/caregivers maintaining involvement in any aspect of adolescent's care

The correct answer is B - adolescent's responsibility for deciding what information can be shared with others Confidentiality is an important concern when providing care for adolescents and young adults. Without reassurances about confidentiality, patients in these age groups may be reluctant to seek out necessary healthcare or to be completely honest and forthright about their needs and concerns. Providing for confidentiality also supports the need for privacy and autonomy. Limits to confidentiality vary from state to state and the provider needs to be familiar with local standards and laws. Not all information can be kept confidential, and it is very important for the provider to discuss this openly with both the patient and their parent or caregiver. It is important to explain that if there is a concern about a danger to self or to others, the provider is required to break confidentiality to address immediate safety needs of the patient or others. For other types of sensitive information, the adolescent or young adult needs to be actively involved in deciding with whom this information can be shared. The provider may encourage patients to share information with family or trusted adults, but ultimately it is the patient's decision.

The procedure for treatment of a wart using liquid nitrogen includes A. applying for 20-30 seconds or until patient complains of discomfort. B. applying until an area 1-3 mm beyond the wart turns white. C. treating weekly until wart is resolved. D. avoiding use on hyperkeratotic warts.

The correct answer is B - applying until an area 1-3 mm beyond the wart turns white. Warts are harmless but resistant growths on the skin and are actually found in children more often than in adults; warts are caused by a form of the human papillomavirus. Most of the time warts resolve and disappear on their own but, occasionally, may need treatment. Treatments range from over the counter products to more invasive or painful options like using liquid nitrogen, electrocautery, or surgical excision. Liquid nitrogen should be applied to the wart for 2 to 10 seconds until an area of 1-3 mm beyond the wart turns white or until the child complains of pain. The goal of liquid nitrogen treatment is to form a blister at the epidermal-dermal junction. Freezing of warts over joints and lateral aspects of digits is more painful and typically not well tolerated by children. Repeated treatment may be needed to be successful. At home treatments include salicylic acid, which is applied to the wart with a toothpick once a day for 4-6 weeks, and duct tape applied for 12 hours a day for 6 days. These treatments have varied success rates and are typically slow to work. They are usually not successful for warts measuring under 5 mm in diameter.

A child presents with a 4 mm, non-tender, swollen, mildly erythematous mass on the central upper eyelid. The MOST likely diagnosis is: A. blepharitis B. chalazion C. dacrocystitis D. external hordeolum

The correct answer is B - chalazion A chalazion is a chronic, sterile, granulomatous inflammation around the meibomian gland. It is usually centrally located along the lid. Although an internal hordeolum (stye) may be located in the same area of the eyelid, clinical findings of the hordeolum include a tender, swollen red furuncle. In contrast, the course of a chalazion includes mild erythema and slight swelling of the lid. After a few days, the inflammation resolves, and a painless mass remains. An external hordeolum or stye is found closer to the lid margin. Blepharitis is inflammation of the eyelid margins and does not present with a mass. Dacrocystitis is an acute infection of the nasolacrimal duct presenting with redness, swelling and tenderness. The nasolacrimal duct is located in the medial lower eyelid area.

A pediatric nurse practitioner is considering a position with a pediatric pulmonologist in a clinic which specializes in the treatment of cystic fibrosis. However, there are patients in the practice who are over the age of 21 years. Which is the BEST action to take? A. decline the job offer for one in which only pediatric patients will be seen B. contact the state board of nursing regarding regulations related to patient populations C. agree to take the job since cystic fibrosis is a pediatric illness regardless of the patient's chronological age D. contact the certification agency to determine their rulings regarding patients over the age of 21

The correct answer is B - contact the state board of nursing regarding regulations related to patient populations Pediatric nurse practitioners (PNP) must base practice decisions on national and state legislation, which is usually guided by recommendations from organizations such as the American Academy of Pediatrics (AAP) and the National Association of Pediatric Nurse Practitioners (NAPNAP). Practice standards and competencies exist for this purpose. Ultimately, however, individual states determine scope of practice and licensure regulations, so the PNP must be familiar with these rulings in order to practice under a state's jurisdiction. Some states require various types of physician collaborative agreements which can spell out specifics such as the age of patients being seen. The AAP and NAPNAP both offer support for pediatric providers caring for older patients with specific pediatric problems. The group of licensing, accreditation, certification and educational organizations known as LACE who have monitored the implementation of the APRN Consensus Model has also written guidance on the topic of age parameters for practice. Before taking any position, it is important for the PNP to examine specific state guidelines and be familiar with practice recommendations from all recognized organizations.

The parents of an otherwise healthy 4 year old diagnosed with immune thrombocytopenia purpura (ITP) are frustrated because they feel "nothing was done" for their child. Physical exam reveals an active, playful child with multiple purpura and petechiae scattered over the skin but no mucocutaneous bleeding. A repeat complete blood count (CBC) shows platelets of 45,000/mm3, hemoglobin of 9 mg/dL, and no abnormalities of the red or white blood cells. The NEXT step includes A. referral for intravenous immunoglobulin (IVIG) infusion. B. discussion regarding safety concerns. C. referral for hospital admission. D. initiation of steroid therapy.

The correct answer is B - discussion regarding safety concerns. Immune thrombocytopenia purpura (ITP) occurs in approximately 4 - 5 children per 100,000 with a peak incidence between the ages of 2 and 5 years. The cause is unknown but thought to be related to immune factors. Expectant management is all that is required for most children and includes regular follow-up visits and repeated blood counts. Complete remission is seen in 89-90% of children within 6 - 9 months. Treatment is generally not indicated unless the course of the illness is chronic, the platelet count is less than 20,000/mm 3 , there is extensive bleeding, or when a protective home environment cannot be provided. Platelet transfusions are generally reserved for children with life-threatening complications such as an intracranial bleed. Although removal of the spleen produces a normal platelet count in 75% of the children with ITP, this surgery is reserved for children with long-term persistent problems due to the risk of fulminant post-splenectomy sepsis. Intravenous immunoglobulin (IVIG) is an alternative treatment for ITP and will induce a rapid rise in platelets, which may then decrease again. It also requires hospitalization and is expensive.

Parents ask about using a storytelling app on a tablet with an interactive screen for their 15 month old. Which of the following guidance is the MOST appropriate? A. caution that prolonged screen exposure can cause decreased visual acuity B. encourage reading and singing directly with the toddler instead of using digital media C. early opportunities to learn using digital media have advantages for future education D. divide toddler's 2 hours of daily screen time into shorter segments

The correct answer is B - encourage reading and singing directly with the toddler instead of using digital media Research shows that young children of toddler age cannot learn information from screens. Children this age learn by interacting with caregivers such as through being read, talked, and sung to, and through exploring their environment. The American Academy of Pediatrics (AAP) recommends that infants and children younger than 18 months not watch television or use digital media. There is no evidence that electronic screens cause harm to a toddler's visual acuity. The AAP recommends limiting screen use for children ages 2 to 5 to just 1 hour a day of high-quality programming.

Which of the following physical activities would be beneficial to a child with stage 1 hypertension? A. weight lifting B. inline skating C. horseback riding D. rock wall climbing

The correct answer is B - inline skating All children and adolescents should be encouraged to participate in regular exercise due to the cardiovascular, pulmonary, and weight maintenance or reduction benefits. In children with hypertension, regular exercise has been shown to reduce both systolic and diastolic pressure. Those with stage 1 hypertension are encouraged to participate in most types of exercise, but caution is needed when recommending exercise to those with stage 2 hypertension until blood pressure is normalized. Aerobic exercises are beneficial in reducing blood pressure. Among the listed activities, inline skating has the highest cardiac demand and is the only aerobic exercise. Listed activities with the lowest cardiac demand are horseback riding and weightlifting. Rock wall climbing is mostly anaerobic exercise but can be aerobic based on the intensity and pace of the movements. Isometric activity is associated with increased blood pressure.

A child is referred to a mental health provider for treatment. To comply with the Health Insurance Portability and Accountability Act (HIPAA) and obtain information about the child's treatment plan, the primary care provider A. must provide a written disclosure agreement from the patient and guardian. B. is allowed access without written authorization from the patient or guardian. C. must provide a written disclosure agreement if a shared electronic health record is not used. D. is allowed access if the mental health provider is not part of the same employer group.

The correct answer is B - is allowed access without written authorization from the patient or guardian. The Health Insurance Portability and Accountability Act (HIPAA) Privacy Rule establishes standards for patient privacy rights and protection of health information. While meeting these essential standards, timely provider-to-provider communication is also important to providing collaborative and integrated patient care. There are often perceived, unnecessary barriers about the exchange of clinical information among care team providers due to misconceptions of the HIPAA Privacy Rule. In most cases, pertinent mental health treatment information can be released to and discussed with members of the current treatment team without written disclosure authorization. As long as both providers are involved in the treatment or consultation of a mutual patient, relevant protected health information can be disclosed without patient or caregiver consent. Written psychotherapy notes and substance abuse treatment records maintained by a licensed substance abuse program are two exceptions that require written disclosure authorization (42 USC §290dd-2; 42 CFR 2.11). It is important to note that individual state laws may impose additional restrictions that supersede HIPAA rules and healthcare providers need to be familiar with variations in states where they practice.

For an adolescent presenting with a scaly, nonpruritic, copper-colored rash that is limited to the torso, palms of hands, and soles of feet, which is the next BEST step? A. obtain a scraped skin culture of the rash B. obtain a nontreponemal test C. prescribe topical steroids D. prescribe oral antifungals

The correct answer is B - obtain a nontreponemal test While non-specific scaly rashes are common in children and adolescents, it is important to differentiate possible causes so that management is appropriate. Scaly patches that are copper in color and involve the torso, palms of the hands, and soles of feet should prompt consideration of secondary syphilis for which the nontreponemal test (RPR) is appropriate. If the nontreponemal (RPR) is positive, a confirmatory treponemal test should be ordered. In 2015-2016, the rate of primary and secondary syphilis cases increased 13% among teens aged 15-19 years. Active nonintervention is appropriate for pityriasis rosea, although oral antihistamines may be used for those who have associated itching. Culturing of scraped specimens is used to test for herpes infection. While topical steroids are effective for eczematous lesions, eczema is rarely found on the palms of hands and soles of feet. Oral antifungal medications may be used in the treatment of tinea versicolor.

A previously healthy 18-month-old female has completed 10 days of an antibiotic for a first febrile urinary tract infection (UTI) and is now asymptomatic. Which is the MOST appropriate NEXT step? A. obtain a bagged urine to determine if UTI has cleared B. obtain a voiding cystourethrogram (VCUG) if a second febrile UTI occurs C. administer daily antibiotic prophylaxis until results of RUS and VCUG are known D. monitor for recurrent infections by urine dipstick testing until imaging results are known

The correct answer is B - obtain a voiding cystourethrogram (VCUG) if a second febrile UTI occurs In 2011, the American Academy of Pediatrics updated the clinical practice guideline for the diagnosis and management of febrile urinary tract infections in children, which was reaffirmed in December 2016. Voiding cystourethrogram (VCUG) is not routinely recommended after the first UTI. However, it is indicated if renal and bladder ultrasonography reveals hydronephrosis, scarring, or other findings suggestive of high-grade vesicoureteral reflux or obstructive uropathy. VCUG should also be performed if there is a recurrence of a febrile UTI. In the reaffirmed guideline, there is no indication for daily prophylactic antibiotic treatment. There is also no recommendation for parents to monitor urine samples at home. Bagged urine samples are not sterile and have a high false-positive rate (63%). They can also become loose, even if applied correctly, and are uncomfortable for the infant when removed. When deciding if an antibiotic should be prescribed, it is important to have accurate test results, and bagged urine samples do not provide the best evidence of an actual UTI.

Appropriate instruction for a young adolescent male considering strength training to enhance muscle mass includes that A. workouts should focus on 1-2 muscle groups on any exercise day. B. powerlifting should be delayed until later in reproductive maturation. C. strength training should occur at least five times a week to be effective. D. other forms of exercise should be avoided on days that strength training is done.

The correct answer is B - powerlifting should be delayed until later in reproductive maturation. Strength training refers to the progressive use of mixed movement speeds to increase muscle strength and stamina. The use of weight machines, free weights, jumping, and jogging all increase stamina and strength. Strength training has been found to be beneficial, even to young athletes as long as it is done safely and supervised by a coach or experienced trainer. Powerlifting should be avoided in prepubescent children due to possible injury to long bones and growth plates as well as the back. Exercises should be balanced among all muscle groups, including strengthening for core muscles. All training sessions should begin and end with a period of warm-up and cool-down exercises that include stretching and dynamic movement such as a slow jog, jumping, or skipping. Athletes should do strength training no more than two or three times a week, and on nonconsecutive days to allow for recovery and to produce the greatest gains in strength building.

A continuous murmur is auscultated on an asymptomatic 2 month old born at 35 weeks gestation. Which of the following is the NEXT step? A. obtain an echocardiogram B. refer to pediatric cardiologist C. obtain an electrocardiogram D. refer to pediatric cardiothoracic surgeon

The correct answer is B - refer to pediatric cardiologist The most common congenital disorder diagnosed in newborns is congenital heart disease. The incidence and mortality of congenital heart disease varies worldwide; recently in the U.S. the number of children with a congenital heart defect has been estimated to be about 1 million infants, children, and adolescents. Murmurs are one of the most detectable clinical features of a congenital heart defect, with auscultation of both murmurs that are benign or innocent and those that are concerning. In addition to obtaining a thorough physical exam to document details about activity, weight gain, respiratory status, pulses, and more, the most important next step would be to refer the infant to a pediatric cardiologist for further evaluation, diagnosis, and management. Studies such as chest radiograph, ECG, ECHO, and CBC will most likely be ordered.

The benefit of fluoride is achieved through which of the following mechanisms? A. bactericidal effect on Streptococcus mutans B. remineralization of carious lesions C. prevention of plaque adherence D. destruction of milk sugars

The correct answer is B - remineralization of carious lesions It is important for healthcare providers to screen for dental caries and to identify risk factors for development of decay. Caries occur when there is overgrowth of the bacteria Streptococcus mutans, Streptococcus sobrinus, and lactobacillus species, which are part of normal human flora. These bacteria adhere well to the tooth surface. Topical fluoride is effective in assisting in the prevention of caries by inhibiting demineralization and aiding in the remineralization of carious lesions. While fluoride does not prevent plaque adherence, it can alter the enamel of teeth making it more resistant to the destructive effects of the bacteria found in the mouth. Milk sugars are a common cause of caries providing a medium for the bacteria to flourish.

When a child has an avulsed permanent tooth, the BEST management includes seeking immediate dental care and A. submerging the tooth in a cup of cold milk. B. replanting the tooth in socket and holding in place. C. submerging the tooth in a cup with the child's saliva. D. soaking tooth in warm water, then wrapping in paper towel.

The correct answer is B - replanting the tooth in socket and holding in place. Primary teeth cannot be replanted. If permanent teeth are knocked out and the tooth is found, timing is important. A tooth that dehydrates may not be successfully replanted after 60 minutes, but dental evaluation is still essential as other interventions may be successful. The best management of an avulsed permanent tooth is replantation in the socket. If the tooth is dirty, it may be cleansed gently under cold running water before replacing in the socket but touching the root area should be avoided. If the tooth can't be replaced into the socket or the caregiver is too uncomfortable about attempting this, the tooth can be placed in cold cow's milk or another isotonic solution such as the child's saliva. Holding the non-replanted tooth in the mouth is a safety hazard with risk of aspiration. Wrapping the tooth in paper towel further dehydrates the tooth and could damage the root.

Developmental milestones in a typically developing 6 month old include A. making sounds that mimic one syllable words, rocking on hands and knees, using a fine pincer grasp. B. rolling from front to back, reaching for an object, and smiling spontaneously at familiar faces. C. sitting unsupported, transferring an object from hand to hand, smiling and cooing. D. crying when parent leaves the room, beginning to creep, and holding onto a toy.

The correct answer is B - rolling from front to back, reaching for an object, and smiling spontaneously at familiar faces. A 6 month old can roll from tummy to back, will reach for an object and bring it to his mouth, and smiles and laughs in response to familiar faces. The 6 month old will sit with support but is not yet adept at transferring objects from hand to hand. The fine pincer grasp is not developed until beyond 9 months of age, which is also when the infant becomes familiar with parents and will recognize when the parents leave the room. Infants begin to creep and rock on hands and knees at an age older than 6 months.

Administration of the 23-valent pneumococcal polysaccharide vaccine is indicated for a 3 year old who has: A. plans for travel to India B. stage 3 chronic kidney disease C. a sibling with human immunodeficiency virus infection D. Down syndrome with a closed ventricular septal defect

The correct answer is B - stage 3 chronic kidney disease The 23-valent pneumococcal polysaccharide vaccine (PPSV23) is recommended for children two years and older with underlying medical conditions that put them at risk for invasive pneumococcal disease caused by serotypes not usually acquired by otherwise healthy children. These medical conditions include chronic heart disease, particularly those associated with heart failure and cyanosis, chronic lung diseases including asthma requiring high steroid doses for treatment, diabetes mellitus, cerebrospinal fluid leaks, cochlear implants, sickle cell disease, hemoglobinopathies, congenital and acquired asplenia, human immunodeficiency virus (HIV) infection, chronic renal failure and nephrotic syndrome, and other illnesses associated with immunosuppressive medications and immunodeficiency. In particular, if cochlear implant or elective splenectomy is planned or immunosuppressive therapy is expected, the PPSV23 should be administered two weeks before the planned procedure. A second dose of PPSV23 is administered five years after the first for those individuals with a condition that causes immunocompromise, including asplenia. There are no indications for this vaccine related to travel. While immunization of groups can offer herd immunity, there are no special indications for this vaccine related to siblings with HIV.

An adolescent sustains a puncture wound in the sole of a foot after stepping on a protruding nail. In addition to reviewing tetanus vaccine status, the MOST appropriate INITIAL management is A. debridement of epidermal edges. B. superficial irrigation of the wound. C. initiation of prophylactic antibiotics. D. probing the wound for retained material.

The correct answer is B - superficial irrigation of the wound. Initial management of puncture wounds includes superficial irrigation with sterile saline and cleansing the area of foreign matter to prevent tattooing of debris into the dermal layers. High pressure wound irrigation as well as surgical debridement and wound probing may damage the tissue and push foreign objects and bacteria deeper into the wound. Prophylactic antibiotics are not indicated unless the risk of infection is especially high such as with cat bites, puncture wounds of the face, and human bite wounds. A tetanus booster is indicated if it has been more than 10 years since the patient's last dose of a tetanus-containing vaccine or if less than 3 doses have ever been received.

A 4 month old with congenital nasolacrimal duct obstruction had several episodes of dacryocystitis and is currently hospitalized for orbital cellulitis. Which is the appropriate follow-up? A. continued frequent massage over the lacrimal duct B. surgical referral for potential tear duct probing C. daily use of an ophthalmic antibiotic ointment D. oral antibiotic prophylaxis

The correct answer is B - surgical referral for potential tear duct probing Congenital nasolacrimal duct obstruction occurs in about 6% of newborn infants and resolves spontaneously in most children by 12 months of age. Symptoms include excessive pooling of tears in the eye, overflow of tears onto the lid or cheek, and reflux of mucoid material from the lacrimal sac. Appropriate conservative management of the condition includes frequent massage and cleansing of the lids and lashes with warm water. Application of an ophthalmic antibiotic ointment or drops is useful when mucopurulent discharge is present. However, ophthalmic antibiotic ointment or drops should not be used on a continuous basis because it may be irritating to the cornea. Gentle downward massage over the nasolacrimal duct is helpful in clearing the accumulation of mucoid material, though it is not established as a curative measure. If tearing persists after 12 months of age, or if the younger infant has repeated episodes of dacryocystitis, earlier referral to an ophthalmologist for tear duct probing or other surgical management is recommended.

The long-term management of a child with simple febrile seizures includes A. bathing in tepid water at onset of fever. B. using antipyretics for the comfort of the child. C. administering acetaminophen or ibuprofen prophylactically at onset of fever. D. administering acetaminophen or ibuprofen 1 hour prior to routine vaccinations.

The correct answer is B - using antipyretics for the comfort of the child. Febrile seizures occur in children between the ages of 6 and 60 months who do not have a history of afebrile seizures, intracranial infection, or metabolic disturbance; the seizure can be attributed to the rapid rise of body temperature, often due to a viral infection. Febrile seizures are classified as either simple or complex. Children who have a simple febrile seizure can have another, and the rate of recurrence varies by age. Children who are younger than 12 months at the time of their first simple febrile seizure have an approximately 50% probability of having recurrent febrile seizures. Other than the likelihood of recurrence, no long-term effects of simple febrile seizures have been identified. Theoretically, any proposed therapy for febrile seizures would need to have minimal risks and adverse effects. The combination of phenobarbital and acetaminophen and the use of continuous valproic acid have been shown to be effective in preventing recurrence, but the adverse effects are not worth the risks. Antipyretics alone make no difference in recurrence of febrile seizures but do provide comfort to the child. No studies have demonstrated that antipyretics, in the absence of anticonvulsants, reduce the recurrence risk of simple febrile seizures.

A school-age child presents with a two-day history of painful, perianal erythema and itching, accompanied by a fever of 100.5 F (38 C). Which of the following is the MOST important history question? A. "Was a fever-reducer given?" B. "Have you changed bath soaps recently?" C. "Has the child had a sore throat recently?" D. "Does anyone in the house have a similar rash?"

The correct answer is C - "Has the child had a sore throat recently?" The child in this scenario is likely presenting with a perianal streptococcal infection. Common symptoms include erythema, pain, local itching, and blood-streaked stools. While presentation with fever can occur, many cases do not involve systemic symptoms. The infection is usually a result of autoinoculation from a recent group A streptococcal infection. On physical exam, the perianal area is erythematous with distinct margins that can extend up to 2 cm from the anus. While the history questions posed in the options are appropriately asked, the most important is whether there is history of a recent sore throat which could cause the streptococcal autoinoculation. Rapid strep test of a perianal swab assists in confirming the diagnosis. Treatment for perianal infection is the same as for streptococcal pharyngitis with penicillin V or amoxicillin being the most appropriate antibiotics. Perianal pruritis is also seen with pinworm infections. However, these children typically present with irritability, hyperactivity, and insomnia. Pinworm infections are not painful and do not present with systemic symptoms. On exam during the nighttime, long, white threadlike worms can be seen at the anus.

An adolescent used a prescribed emergency contraceptive after having unprotected sexual intercourse. A return visit and pregnancy test should be advised if there is no menstrual period within: A. 1 week B. 2 weeks C. 3 weeks D. 4 weeks

The correct answer is C - 3 weeks An emergency contraceptive can be prescribed for adolescents who are sexually active to prevent a pregnancy in the event that a condom breaks, no contraceptive was used during vaginal intercourse, or the contraceptive was forgotten. Some references advise a preemptive emergency contraception prescription to be written for all female adolescents who are sexually active. There are limited choices for medications for emergency contraception. It is recommended that they be taken as soon as possible after unprotected sex and are most effective if taken within 72 hours. Guidance should include that after taking an emergency contraceptive, the adolescent should return for a pregnancy test within 3 weeks if menses have not started. In the meantime, the adolescent should use condoms or refrain from sexual intercourse until the next cycle starts. Emergency contraceptives may also alter the schedule of menses but, in general, menses will likely come sooner than expected. Although ectopic pregnancy is rare, guidance should also include the importance of seeking immediate evaluation should severe abdominal pain occur 3-5 weeks after use of emergency contraception.

Caregivers are concerned about a milk protein allergy in their 2 week old due to an older sibling's allergies. They are currently feeding standard cow's milk formula and want to switch to a partially hydrolyzed cow's milk formula. Which is the BEST information to share? A. partially-hydrolyzed formula does not contain adequate amounts of protein for infants under 1 month B. partially-hydrolyzed formula does not contain adequate amounts of iron for infants under 1 month C. both formulas contain equal nutrients including iron. D. both formulas contain the same form of milk protein

The correct answer is C - both formulas contain equal nutrients including iron. As outlined in federal regulations, all U.S. produced formulas contain equal nutritional value as long as they are designated for infant feeding. The difference between partially hydrolyzed formulas and standard formulas is the milk protein is partially "digested" or broken down to oligopeptides with a lower molecular weight. Both types of formulas contain the recommended amount of iron and nutrients, which is regulated by the FDA. Partially or extensively hydrolyzed formulas are recommended for use when there is a concern for developing milk protein allergy or risk for atopic disease. If true milk-protein allergy symptoms occur in this infant, extensively hydrolyzed formula or amino acid formula would be recommended.

Which scenario BEST represents an example of child ASSENT in health care research? A. 5 year old who passively shrugs when asked for permission to participate in a clinical trial B. 6 year old who is shown pictures of the research opportunity and agrees with a nodding of the head C. 9 year old who verbally refuses then consents to participate the following day D. 10 year old who is asked by the parents to participate and is silent when approached

The correct answer is C - 9 year old who verbally refuses then consents to participate the following day In healthcare and healthcare research, the terms "assent" and "consent" have distinct meanings and represent different levels of understanding and agreement. In most situations, legal consent for procedures or participation in research, for example, can only be given by an adult or an adolescent who meets requirements to have been emancipated. Assent, or the child's permission to participate, is influenced not only by the child's chronological age but perceived developmental understanding. Thus, the capacity to engage in assent is best described as on a continuum, with a positive assent dependent upon the child's developmental level and capability. The younger child, due to developmental limitations, cannot be viewed as an independent decision maker despite informational methods used. Additionally, assent should be an affirmative choice and not simply silence or passive resignation. When seeking child assent, the child's choices should be taken seriously. Therefore, the child's dissent should also be respected

The parent of a 12 month old being seen for a well-child visit has epilepsy. Based on recommendations by the Advisory Committee on Immunization Practices (ACIP) for administration of primary doses of the MMR and varicella, the infant should be administered the A. MMRV today. B. varicella vaccine today and the MMR in 2 weeks. C. MMR and varicella today as separate vaccinations. D. MMRV when the child is 4 years of age.

The correct answer is C - MMR and varicella today as separate vaccinations. Vaccination for protection against measles, mumps, rubella (MMR) and varicella is a 2-dose series with the primary dose given at 12-15 months of age. If the MMR vaccine is administered for travel before 12 months of age, it is not counted toward the 2-dose schedule. The combination vaccine MMRV can be used for both doses; however, the MMRV vaccine is associated with the possibility of a subsequently higher fever and a two-fold increase in febrile seizures as compared to MMR and varicella administered as separate vaccinations on the same day. Because of this risk, the Advisory Committee on Immunization Practices (ACIP) recommends separate MMR and varicella vaccines for the primary dose unless the caregiver understands and prefers the MMRV. For children with a personal or family history of seizures, the ACIP recommends separate vaccination on the same day for both doses of the series. Contraindications to MMR and varicella vaccines include pregnancy, untreated tuberculosis, immunocompromised state, or anaphylaxis to any component of the vaccines. Live vaccines, if not administered on the same day, need to be separated by at least 30 days to be valid and provide protection. If for some reason a 4-year old has never been vaccinated against MMR and varicella, and there is no personal or family history of seizures, the child may receive the MMRV for the primary dose as the occurrence of a first febrile seizure after 4 years of age is uncommon.

Which of the following is the MOST appropriate information to provide to a pregnant 16 year old whose immunizations were last updated at age 12? A. Tdap vaccine is not recommended during pregnancy. B. Tdap vaccine is recommended every 10 years, the next dose should be given in 5 years. C. Tdap vaccine is recommended with every pregnancy. D. After the initial Tdap series, only the tetanus toxoid vaccine is recommended every 10 years.

The correct answer is C - Tdap vaccine is recommended with every pregnancy. The Advisory Committee on Immunization Practices (ACIP) published revised recommendations for the Tdap vaccine in 2013. In 2011, the ACIP previously recommended that unvaccinated pregnant women receive the vaccine; the update now recommends Tdap with each pregnancy regardless of previous history. During several years prior to this recommendation, substantial increases in the cases of infant pertussis occurred, thus prompting the new recommendations. Newborn and very young infants are unable to mount a cell-mediated response and rely solely on maternal antibody for protection from pertussis. Vaccinating the pregnant woman stimulates development of maternal antibodies against pertussis, which are then transferred across the placenta to the fetus. The mother is also protected around the time of delivery, decreasing her risk of contracting pertussis and transmitting it to the infant. Tdap may be administered at any time during pregnancy but is preferred during the third trimester to provide the greatest transfer of maternal antibody nearer to the time of birth. Studies in nonpregnant adolescents and adults have not shown any serious adverse events in persons who received Tdap and tetanus and diphtheria toxoids (Td) or Tdap-inactivated polio vaccines at short intervals between doses. These studies include small numbers of persons and, thus, may not identify rare but significant adverse events. No significant adverse events have been noted in large studies of tetanus toxoid use during pregnancy. Tetanus toxoid and Td have been used extensively worldwide in pregnant women for prevention of neonatal tetanus. There is a theoretical risk for severe local reactions for pregnant women who have multiple pregnancies within short intervals, but the evidence suggests no increased risk of severe adverse effects for women receiving Tdap with every pregnancy. The ACIP believes that the morbidity and mortality associated with pertussis in infants outweighs the possible theoretical concerns of severe adverse events.

For a toddler with poor weight gain, which of the following questions in MOST appropriately asked when obtaining the history of present illness from the caregiver? A. Was your child born at full-term? B. Is there a family history of celiac disease? C. What has your child eaten in the past day? D. How long was your child breast or formula-fed?

The correct answer is C - What has your child eaten in the past day? The history of present illness (HPI) is one component of a comprehensive or symptom-focused visit. The HPI questions that a provider asks will focus on the current situation and seek details about onset, associated symptoms, characteristics, duration, exposures, current home management, and similar occurrences in the past. A comprehensive health history, on the other hand, includes prenatal and birth history, past medical history, family history, and review of systems. The history for a symptom-focused visit includes only components likely to contribute to diagnosis and treatment. The gestational age and the duration of breast/formula feeding provide prenatal and past medical history. A 24-hour recall provides detailed information about the present caloric and nutritional intake that may be contributing to poor weight gain. While the presence of celiac disease in the family history is pertinent, this is initially less important than obtaining details about caloric intake.

An adolescent presents for evaluation of chest pain that occurs during gym class, with episodes happening several times over the past month. Which of the following HPI components is MOST specific for differentiating a cardiac etiology? A. resolves with rest B. location is midsternal C. associated with syncope D. quality is sharp and stabbing

The correct answer is C - associated with syncope Chest pain associated with syncope, exertional dyspnea, or irregularities in heart rhythm needs careful evaluation for a cardiac cause. When pain is of cardiac origin, the specific history and details given by the child or adolescent is usually consistent across events. Midsternal pain can indicate gastroesophageal reflux. Any burning substernal pain that worsens with reclining or with spicy foods are most likely of gastrointestinal etiology. Pain that is sharp or stabbing, lasting several seconds to minutes, located over the mid-sternum or infra-nipple area, and occurring with non-exertion or deep inspirations are more likely musculoskeletal in origin.

In addition to hypoglycemia, the MOST common complications found in infants born to mothers with diabetes or gestational diabetes include: A. cardiomegaly, hypotonia, and polycythemia B. anemia, hepatosplenomegaly, and hypotonia C. cardiomegaly, polycythemia, and respiratory distress D. anemia, neural tube defects, and respiratory distress

The correct answer is C - cardiomegaly, polycythemia, and respiratory distress About 36% of infants born to mothers with diabetes or gestational diabetes are generally large for gestational age (LGA) and predisposed to many anomalies and metabolic disorders related to diabetic control throughout the pregnancy. Hypoglycemia develops in about 27% of these infants, with only a small percentage actually demonstrating symptoms of jitteriness, hyperexcitability, and poor feeding. Five to 8% have major congenital malformations. Cardiomegaly is common in infants of diabetic mothers, occurring about 30% of the time and is associated with anomalies such as asymmetrical septal hypertrophy, subaortic stenosis, atrial septal defect, and transposition of the great arteries. Additional anomalies can include neural tube defects and defects of the GI tract and kidneys. These infants have a higher risk of respiratory distress (34%) and an increased incidence of hyperbilirubinemia, renal vein thrombosis, and polycythemia (5%).

A 12 month old has a normal white blood cell count and normal platelet count, with the following additional results (see table below). After the infant is started on iron supplementation, the MOST appropriate laboratory follow-up plan is to obtain a: A. serum ferritin in 2 weeks B. serum iron levels in 2 weeks C. complete blood count in 1 month D. total iron binding capacity in 1 month

The correct answer is C - complete blood count in 1 month Iron deficiency anemia is a common nutritional disorder and frequently affects children 12-36 months of age who are given excessive dairy products and insufficient intake of high-iron foods. Persistent iron deficiency anemia can negatively affect cognitive function and learning. Severe anemia can contribute to fatigue, irritability, and a change in the oxygen dissociation curve. The American Academy of Pediatrics recommends that a child's hemoglobin level be checked at 12 months of age. The low hemoglobin and hematocrit and reduced mean corpuscular volume (MCV) indicate that the child in this scenario has iron deficiency anemia likely related to an insufficient dietary intake of iron. The first laboratory indication of iron deficiency anemia is reduced iron stores, which is reflected in the ferritin value, a protein that estimates the stores of iron. As the stores of iron decrease, the capacity to absorb more increases, which is evaluated through total iron binding capacity. With reduced availability of hemoglobin, the size of red blood cells (RDW) increases and the MCV decreases. Administration of iron supplementation will increase the hemoglobin by 1-2 g/dL within a month. Supplemental iron is continued for 2-3 months after normalization of the hemoglobin to develop adequate iron reserves. With severe anemia, a reticulocyte count can be obtained shortly after iron therapy has begun to confirm the diagnosis of iron deficiency anemia. Serum iron, ferritin, and total iron binding capacity are not required to make the diagnosis of iron deficiency anemia and are usually obtained when the hemoglobin and hematocrit are not improving as expected with iron supplementation. Serum ferritin levels may be tested 6 months after iron supplements are stopped to assure adequate iron stores.

An adolescent reports daily headaches for the past 6 months that are immediately preceded by nausea and changes to vision in the right eye, seen as wavy lines. The patient self-medicates by taking Ibuprofen with each headache but only gets partial pain relief. The neurological exam is normal. What is the BEST NEXT step in management? A. obtain MRI B. prescribe sumatriptan C. discontinue analgesics D. recommend feedback therapy

The correct answer is C - discontinue analgesics Migraine is the most common headache in childhood and may have an aura associated at the onset of the headache. Auras are typically visual symptoms but may also be sensory, occurring less than 60 minutes before the start of the headache. There is a familial predilection for migraines. Migraine headaches are typically throbbing and have associated nausea, vomiting, and light and/or sound sensitivity. Common triggers are disordered or insufficient sleep, dehydration, and skipping meals. Ibuprofen is an effective analgesic in the treatment of migraines, but analgesic use should be limited to no more than 3 uses per week. Medication-overuse headaches are caused by using analgesics too frequently. Use of analgesics more than 3 times per week for headaches can actually cause headaches. Treatment of medication-overuse headaches requires discontinuation of the analgesics. If there are neurological abnormalities on examination or a history of headaches that awaken the individual from sleep, an MRI is the most appropriate imaging strategy. Triptan medications can be an effective abortive therapy for migraines and feedback therapy can be effective in acute and preventative management to provide the individual with some self-control. Focus should be on good life-style activities including regular and sufficient sleep, avoidance of skipping meals and ensuring adequate protein intake, minimizing caffeine intake, and increasing hydration. After a month of no analgesic use and focusing only on life-style activities, re-evaluation of the headache frequency is warranted, at which time an appropriate treatment plan can be created that includes limiting the frequency of analgesic use.

A school-age child with phenylketonuria has started eating school cafeteria lunches with friends to include eating dairy products and meats. This dietary nonadherence increases the child's risk for: A. anaphylaxis B. bipolar disorder C. hyperactivity D. lactose intolerance

The correct answer is C - hyperactivity Phenylketonuria (PKU) is a genetic disorder characterized by a mutation in the phenylalanine hydroxylase gene, the gene responsible for converting the amino acid phenylalanine to tyrosine. This gene mutation can lead to an excessive buildup of phenylalanine, which interferes with the developing brain. A phenylalanine-restricted diet is the primary treatment for PKU. Foods with high amounts of phenylalanine include protein-rich foods like meat, eggs, milk, and peanut butter. These foods must be avoided. Nonadherence to the phenylalanine-restricted diet can cause neurological changes that can result in cognitive deficits, developmental delays, seizures, EEG changes, and behavioral changes to include hyperactivity, irritability, distractibility, anxiety, and depression. While treatment can reverse behaviors such as hyperactivity, it will not reverse resultant intellectual disabilities. 1 Individuals with PKU also require daily supplementation of phenylalanine-free elemental medical foods for nutritional support. Most state laws require that insurance companies cover the cost of these foods. When diagnosed and treated early with appropriate dietary interventions, children with PKU can progress in school like their peers.

For an adolescent's self-management of dysmenorrhea, which analgesic is most appropriate with the LOWEST risk of gastrointestinal adverse events? A. acetaminophen B. aspirin C. ibuprofen D. ketorolac

The correct answer is C - ibuprofen Non-steroidal anti-inflammatory drugs (NSAIDS) provide the most effective management of menstrual cramps as they decrease prostaglandin production, which causes the pain. Acetaminophen, while not having gastrointestinal (GI) adverse events, it is not an NSAID, so it is not as effective for dysmenorrhea. While ketorolac has the lowest GI adverse events, it is administered intravenously and, therefore, not an appropriate medication for management of painful menses. Aspirin has the greatest risk of bleeding and GI complications. Ibuprofen is safe for short-term use and effective for management of painful menses. It has a low rate of serious GI complications with short term use but should be avoided for individuals with underlying bleeding disorders.

A school-age child has an asthma treatment plan using fluticasone 44 mcg inhaler, 2 puffs twice per day, and an albuterol inhaler 4-6 puffs every 4-6 hours as needed. The child now presents with a second asthma exacerbation in 2 months. What is the best NEXT step? A. prescribe different inhaled corticosteroid B. assess environmental triggers C. inquire about inhaler usage D. obtain chest radiograph

The correct answer is C - inquire about inhaler usage Improvement, control, and resolution of illness is not achieved if the treatment plan is not followed. Nonadherence to medications can contribute to increased use of health care services and lead to complications. Factors that can contribute to non-adherence with medication therapy include the expense of medications, lack of understanding about the purpose of medication(s) or how the medication(s) should be taken, family beliefs about medication, and the perceived severity of the illness. Because adherence to medication regimens for chronic illnesses decreases over time, adherence should be proactively assessed at each health care encounter. Validated questionnaires to assess adherence and non-judgmental inquiries about the use of the medications are effective and may reveal prohibitive costs, lack of understanding, or a need for assistance with identifying creative ways to remember to take medication. Asthma is a chronic illness that can be episodic or persistent. The home treatment plan indicates that this child has persistent asthma, which is best managed by daily controller medication (fluticasone) with episodic use of the rescue inhaler (albuterol). Families often confuse which medicine is the controller versus the rescue medicine and they may not understand that the controller medication is needed even when there are no symptoms. While assessment of environmental factors can help identify new triggers, a different inhaled corticosteroid may be beneficial, and a chest radiograph may identify pulmonary issues needing other treatment, the first step is to always assess if the prescribed plan is being used. Evaluation of when and how each inhaler is used is helpful in differentiating whether the current treatment plan is not being followed versus being ineffective in controlling the asthma, leading to the recent exacerbations.

An adolescent with a history of severe motion sickness is concerned about an upcoming family vacation by car. Which of the following medications is the MOST appropriate recommendation? A. aprepitant B. hydroxyzine C. meclizine D. ondansetron

The correct answer is C - meclizine Meclizine is an antiemetic and antihistamine approved by the FDA for prevention and treatment of motion sickness and management of vertigo. No clinical studies have been done to establish safety or effectiveness in children and, therefore, it is not recommended for children under the age of 12 years. It is available over the counter in 12.5 mg and 25 mg dosages in oral formulations including tablets and chewable tablets. Aprepitant is a substance P/ neurokinin 1 receptor agonist that is FDA approved for prevention of acute and delayed nausea and vomiting in ages 18 years and above. Although it has no approvals for pediatric patients, it is used off-label in children for prevention of chemotherapy-induced nausea and vomiting. It has been used in this clinical setting in both tablet form as well as an extemporaneous preparation of an oral solution. Hydroxyzine is an anxiolytic, antiemetic, antihistamine, and sedative. In its oral form, it is approved by the FDA for use in treatment of anxiety, pruritus, and preoperative sedation in pediatric patients with no ages specified. Ondansetron is an antiemetic in the 5-HT3 receptor antagonist family. Its oral formulations are FDA approved for prevention of nausea and vomiting associated with moderately and emetogenic chemotherapy in children ages 4 years and up. In parenteral formulation, it is approved for prevention of nausea and vomiting associated with emetogenic cancer chemotherapy in infants 6 months of age and older through adults, and in prevention of postoperative nausea and vomiting in infants 1 month of age through adults. Although nausea and vomiting are symptoms of motion sickness, ondansetron is not approved for this use.

A caregiver expresses concern for her 12 month old's lack of interest in playing with other children, especially due to a family history of autism spectrum disorder. Which of the following is the MOST appropriate next step? A. suggest the caregiver attempt to engage the child in imitating behaviors B. screen for social interactions such as peek-a-boo and pat-a-cake C. offer reassurance that interactive play begins at age 3 D. refer for a developmental evaluation

The correct answer is C - offer reassurance that interactive play begins at age 3 Play is an essential component in assessing a child's development. Because there is a slight increased risk of autism spectrum disorder in familial studies, the caregiver is appropriately concerned for the child's risk. Peek-a-boo and pat-a-cake are developmentally appropriate engaging behaviors at this age; however, this does not address the concerns of the caregiver. Imitating behaviors begin between ages 2-3 years, so the child is too young to be evaluated for this type of play. Despite family concerns, a developmental evaluation is not needed as the child's lack of interactive play does not warrant a referral at 12 months of age. By educating about age-appropriate types of play, and implementing consistent developmental screening tools with parental education, the specific concerns are being addressed.

During a routine well visit, a 16 year old discloses illicit substance use. Disclosure of this information to parents/guardians is A. permitted as the substance use is illegal. B. not permitted due to requirements of confidentiality. C. only permitted if substance use poses an immediate safety risk. D. only permitted if the adolescent first signs a formal consent form.

The correct answer is C - only permitted if substance use poses an immediate safety risk. Illicit drug use is a significant issue facing adolescents in the United States. It is estimated that by the time students graduate high school, 7 out of 10 have tried alcohol, more than half have tried an illicit drug, and one-fourth have tried an illicit drug other than marijuana. Primary care providers play an important role in preventing substance abuse and in providing guidance and treatment for those who are engaging in illicit substance use. As part of routine health care for all adolescents and young adults, the AAP recommends annual screening for substance use and brief interventions based on the outcome of short screening questions. The interventions range from positive reinforcement for no substance use to referral when severe substance use is detected. Any questions regarding drug and alcohol use should be asked in private without a parent or guardian present. Before beginning this discussion, it is important to discuss confidentiality and how disclosures will be handled. Adolescents should be reassured that their answers and any discussion that takes place will be kept confidential unless there is a concern about their safety or the safety of others. Whether or not a specific, disclosed behavior poses a safety risk is a matter of clinical judgement. The provider must take into account the behavior disclosed, the patient's age, social situation, and any other diagnoses the patient may have. If the decision is made to disclose the reports of substance use to parents, it is important to discuss with the adolescent beforehand precisely what information will be disclosed, as well as whether the adolescent wishes to be present during this disclosure. It is usually appropriate to focus on diagnosis and treatment during these discussions. Patients who are 18 years of age or older must formally consent to include parents in their medical care including disclosure of substance use.

A parent is concerned about how to address the needs of 4-year-old son who has expressed a desire to be a girl. Which guidance is MOST appropriate? A. prevent dressing in feminine clothes B. promote group activities with other boys C. discourage play with gender-targeted toys D. encourage participation in activities of interest

The correct answer is D - encourage participation in activities of interest Gender non-conformity in children is part of normal development, although only a minority experience persistence into adolescence. Acceptance of the child and management of stigma are most helpful to the child as he/she explores this. Pressure to suppress non-conforming feelings and to participate in activities that society deems appropriate for gender contributes to low self-esteem and increased anxiety. Bullying and social isolation is common to children who are gender non-conforming. Allowing the child to dress in feminine clothes at home, a safe and accepting place, helps the child with self-expression without increasing the risk of social ostracism. Supporting the child's interests and strengths promotes self-esteem and demonstrates acceptance of the child.

A school-age child is seen for a skin rash that began about 6 weeks ago along with bilateral limping, and a recent 2-week history of fever. Examination reveals inflammation of knee joints. Which is the MOST likely diagnosis? A. acute rheumatic fever B. dermatomyositis C. systemic juvenile idiopathic arthritis D. systemic lupus erythematosus

The correct answer is C - systemic juvenile idiopathic arthritis. Juvenile idiopathic arthritis (JIA), previously referred to as juvenile rheumatoid arthritis (JRA), is the most common type of arthritis in children. The exact etiology of JIA is unknown but likely due to an environmental influence in genetically pre-disposed individuals. There are six JIA subtypes. Systemic JIA is considered an autoinflammatory disease, whereas the other types are considered autoimmune diseases. Systemic JIA causes inflammation in one or more joints for 6 weeks' duration in a child younger than 16 years of age and is often accompanied by fatigue, inflammation of joints, skin rash, and a high fever (103° F [39.4° C] or higher) that lasts at least 2 weeks. Pericarditis, pleuritis, anemia, or enlargement of lymph nodes, liver or the spleen may also occur. Researchers believe there is a genetic component to JIA and a virus may trigger the disease process. Systemic lupus erythematosus (SLE) is a chronic systemic rheumatic disease. Symptoms include fatigue, skin rashes, fever, and joint involvement with non-deforming arthritis but with effusion and tenderness. SLE can involve many organ systems (kidney, skin, blood cells, and nervous system). A malar rash across the cheeks and bridge of the nose in the shape of a butterfly is seen in 95% of SLE cases. The most common manifestation of acute rheumatic fever is polyarthritis of large joints. The latency period from infection with group A streptococcal infection until the onset of arthritis is about 2-6 weeks. Acute rheumatic fever is most common in children 5-15 years of age. Dermatomyositis is an autoimmune myopathic illness affecting the pediatric population. Proximal muscle weakness in addition to thick scaling on extensor surfaces, known as Gottron papules, are characteristic of dermatomyositis. Results of laboratory studies are often normal at the time of diagnosis. A definitive diagnosis is made with muscle biopsy; ongoing management is coordinated with rheumatology specialists. Without adequate treatment, muscle weakness will often continue to progress. Early recognition of Gottron papules is key to the diagnosis of dermatomyositis.

Emollients have been ineffective for a child who has psoriasis with involvement of the eyelids. Which of the following is the BEST choice of topical medication? A. calcipotriene B. salicylic acid C. tacrolimus D. tazarotene

The correct answer is C - tacrolimus Topical therapies for psoriasis can vary by the location or skin thickness. In general, the lower potency topical corticosteroids (TCS) are used in areas of thinner skin such as the face, eyelids, flexures and folds, while the higher potency TCS are used for areas of thicker skin. The eyelids or face are at increased risk for adverse effects from long-term steroid use, so the appropriate agent is tacrolimus. Calcipotriene is a vitamin D analogue. These agents are better suited for treatment in areas of thicker plaques including the scalp, extremities, palms and soles. Salicylic acid is a keratolytic, a peeling agent used to debulk thick, localized plaques on the scalp, palms and soles. Tazarotene is also reserved for thicker plaques away from intertriginous sites.

Which of the following is the MOST appropriate phrasing to begin a motivational interview with an adolescent who is obese? A. "What is one thing you think you could change about your lifestyle?" B. "Your blood pressure is elevated and you seem nervous. How are you feeling?" C. "What do you already know about ways to improve your physical health and lose weight?" D. "Your blood pressure is a little high and that worries me. Can you tell me a little bit about your eating habits?"

The correct answer is D - "Your blood pressure is a little high and that worries me. Can you tell me a little bit about your eating habits?" Obesity can be a difficult condition to treat, with patients and their parents often appearing to lack sufficient motivation to make significant changes in eating behaviors, dietary composition, and physical activity. Motivational interviewing uses a nonjudgmental, supportive mode of communication between the healthcare provider (HCP) and patient, designed to enhance motivation for behavior change rather than the more traditional emphasis on the transfer of information about a healthy diet or exercise. The steps in motivational interviewing include: establishing rapport with the patient, setting an agenda, identifying the patient's ability to change, and establishing a plan. When beginning a motivational interview, developing rapport allows the HCP and patient to establish a trusting relationship, which includes active listening and thoughtful reflection. The HCP must engage in active reflective listening to respond effectively to the patient's or parent's questions and statements. Once the patient or parent acknowledges both concern about the blood pressure and a belief that behavior change will be effective in overcoming the problem, it then becomes much more likely that goals for behavior change can be articulated, agreed on, and ultimately achieved.

In a 10 month old, which oral behavior BEST demonstrates a readiness to advance to softer solid foods? A. uses rotary movements to chew meat B. sucks in anticipation of the spoon C. controls liquid taken from cup D. controls bite of soft cookie

The correct answer is D - controls bite of soft cookie During the first several years of life, development progresses for eating and language, both of which call upon coordinated oral movements. Between the ages of 4-6 months, children begin sucking movements in anticipation of the spoon. Children between 10-12 months of age can control bites of soft cookie and begin to use rotary chewing movements but have not yet accomplished this task with meat. By age 16-18 months, the mouth can control liquids taken from a cup, and by 19-24 months the child can chew meat with rotary movements.

An adolescent presenting for a pre-college physical describes herself as a sexually active lesbian. Which is the MOST immediate action? A. assess for potential psychosocial difficulties B. ensure human papilloma virus vaccination is up-to-date C. provide confidential and nonjudgmental care D. counsel about sexually transmitted infections and contraception

The correct answer is D - counsel about sexually transmitted infections and contraception A meta-analysis published in the International Journal of Obstetrics and Gynaecology (BJOG) reported that adolescent lesbian and bisexual females are more likely to have been pregnant when compared to their heterosexual peers. A CDC survey among adolescents regarding sexual identity and sexual contacts further conferred that some respondents who self-identified as heterosexual, only had sexual contact with same sex partners while others who self-identified as lesbian, gay, or bisexual (LGB) had either no sexual contact or only had heterosexual sexual contact. Thus, LGB adolescents and young adults are at greater risk for unplanned pregnancy and, it is important to counsel about accidental pregnancy even with females who have sex with females. Providing sexual and reproductive health education that includes information about abstinence, contraception, and sexually transmitted infections (STIs) is necessary for all adolescents and young adults. Additionally, information about contraception should be offered to all females regardless of their sexual orientation. Pre-college visits present an excellent opportunity to ensure that all immunizations are current, not just the human papilloma virus vaccine. Adolescents and young adults, including those who know or are unsure about their sexuality, may have psychosocial concerns. Providing care that is confidential, current, factual, and nonjudgmental is required with all patients, regardless of their sexual orientation.

Which of the following findings in a 12 month old would warrant a referral for a more thorough developmental evaluation? A. cruises but does not walk independently B. does not use a sipping cup or spoon C. cries when approached by stranger D. favors use of the right hand

The correct answer is D - favors use of the right hand Developmental evaluation should be performed at each well child check according to established developmental guidelines. Gross motor development follows a predictable pattern from the central nervous system's head to toe maturation. Gross motor delays are fairly common in children and range from mild to severe; some will self-correct without intervention while other ones will need intervention. After a gross motor delay is identified, evaluation and treatment may be warranted. Mild delays may be closely surveilled in the primary practitioner's office, whereas other delays such as regression of motor skills or loss of strength require expert evaluation and referral. Hand dominance is usually not exhibited before 18 months of age. Early hand preference may indicate weakness of the contralateral extremity, and in a child less than 1 year of age, may be suspect for cerebral palsy. Children usually walk by 9-15 months of age. Children exhibit stranger anxiety between 6- 18 months of age. Children often do not use household items appropriately until 15-18 months of age; therefore, a 12 month old may not yet use a sipping cup or a spoon.

The evaluation of a child with a suspected learning disability should always include: A. Denver II developmental assessment B. intelligence quotient measurement C. genetic and chromosomal testing D. full history and physical exam

The correct answer is D - full history and physical exam In 2006, the American Academy of Pediatrics recommended developmental surveillance at every well child visit and developmental screening at defined ages. Historically, only about 30% of children with developmental and behavioral problems are identified by their primary care providers. Along with the emphasis on completing this screening, it is most important to initially rule out medical problems related to vision and hearing. Deficits in these areas can affect learning for a child without disability and make good outcomes more difficult for those with developmental disorders. A full history and physical exam are foundational to and must be included with any evaluation for learning, developmental, or behavioral problems.

For a 2 month old with torticollis and moderate lateral deformational plagiocephaly, which of the following is the MOST appropriate action? A. order cranial orthotic therapy B. implement repositioning routine only C. proceed with watchful waiting and revaluate in 1 month D. initiate repositioning routine and refer to physical therapy

The correct answer is D - initiate repositioning routine and refer to physical therapy The optimal treatment of mild to moderate deformational plagiocephaly (DP) in infants younger than 4 months of age is repositioning and physical therapy to address any torticollis. Caregiver education is important and should include recommendations for repositioning and encouraging tummy time for 10-15 minutes when the infant is awake, at least 3 times a day. Also neck stretching exercises with each diaper change can improve torticollis. It is important to reinforce that prone positioning while awake is also beneficial to motor development. The prone position is not recommended during sleep due to its associated risk for sudden infant death syndrome. When placing the infant to sleep, alternating the position from the head or foot of the crib allows the infant to sleep facing into the room with equal time on both sides of the occiput. Cranial orthotic therapy with the use of custom helmets has been shown to be effective in the treatment of infants older than 4 months of age for severe DP and in worsening mild or moderate DP. For infants with severe DP, the use of helmets can be implemented before 4 months. Helmet therapy is usually required for 7 to 8 months and the helmet is worn more that 20 hours a day. Helmet therapy is expensive and may have side effects such as skin irritation and pressure sores.

A 16 year old screens positive for alcohol use but not substance use disorder. Which approach is likely to be MOST beneficial? A. educate regarding the harmful effects of alcohol on the adolescent brain B. refer to community law enforcement program focused on drug prevention C. emphasize consequences of use such as loss of driver's license and school failure D. provide a brief statement normalizing abstinence and explore substitute activities

The correct answer is D - provide a brief statement normalizing abstinence and explore substitute activities Brief interventions by healthcare providers ranging from a few seconds to several hours can significantly reduce alcohol and drug use. Positive reinforcement from a primary care provider for abstaining from substance use can reduce the odds of alcohol use within 3- 12 months after the primary care appointment. It's important not to suggest that it is behaviorally normal to drink at this age and the provider needs to reinforce this, especially with younger adolescents. A strength-based approach is recommended for risk reduction in adolescents where the provider emphasizes skills, talents, and abilities to make healthy choices. Prevention interventions which have been proven ineffective are those based on providing knowledge of harmful effects of substance use and programs featuring authority figures such as law enforcement

Which of the following is the minimum period recommended by AAP in their general guidance for pediatric medical record retention, recognizing that other regulations and rules may apply? A. until the patient reaches the age of majority. B. until the patient is 21 years of age. C. for 1 year after the child is no longer a patient of the practice. D. until the age of majority plus period of state's statute of limitations.

The correct answer is D - until the age of majority plus period of state's statute of limitations. While laws related to medical record retention may differ state to state, many are based on statues of limitations and the recognized age of majority, which is age 18 years of age in most states. Both federal regulations and The Joint Commission may have additional requirements for record retention. The general guidance provided by the American Academy of Pediatrics is that records be retained until the patient reaches the age of majority plus the period of the state's statute of limitations. The average period for statute of limitations for medical malpractice claims is 2 years; however, some state-specific variances allow for periods up to 10 years. For a state with a 2-year statute of limitations period that recognizes the age of majority as 18, a child who was last seen by the practice at 12 years of age would need their records retained for a period of 8 years. Practices must deal with regulatory requirements for the storage of paper and electronic medical records (EMR). EMR requirements include specifications for data backup, encryption, and disposal.


Conjuntos de estudio relacionados

Performance Management and Appraisal

View Set

Operator 1 chapter 6 Trickling filters

View Set

Total Health Chapter 7 - Stress and Anxiety

View Set

Chapter 12: Behavioral Emergencies

View Set

Salesforce Marketing Cloud Administration Exam

View Set

Introduction to Career Skills in Data Analytics

View Set

Chapter 14 - Miscellaneous Commercial Lines Coverage

View Set

DHS 1110 01---Introduction to Health Professions Part 2

View Set

Программирование

View Set

Civil Liberties and Rights Assessment

View Set